Last visit was: 23 Apr 2024, 23:41 It is currently 23 Apr 2024, 23:41

Close
GMAT Club Daily Prep
Thank you for using the timer - this advanced tool can estimate your performance and suggest more practice questions. We have subscribed you to Daily Prep Questions via email.

Customized
for You

we will pick new questions that match your level based on your Timer History

Track
Your Progress

every week, we’ll send you an estimated GMAT score based on your performance

Practice
Pays

we will pick new questions that match your level based on your Timer History
Not interested in getting valuable practice questions and articles delivered to your email? No problem, unsubscribe here.
Close
Request Expert Reply
Confirm Cancel
SORT BY:
Date
Tags:
Show Tags
Hide Tags
User avatar
Manager
Manager
Joined: 23 May 2008
Posts: 224
Own Kudos [?]: 1691 [36]
Given Kudos: 14
Send PM
Most Helpful Reply
Magoosh GMAT Instructor
Joined: 28 Dec 2011
Posts: 4448
Own Kudos [?]: 28569 [25]
Given Kudos: 130
General Discussion
Manager
Manager
Joined: 07 Aug 2010
Status:Now or never
Posts: 249
Own Kudos [?]: 472 [0]
Given Kudos: 27
Location: India
Concentration: Strategy, Technology
GPA: 3.5
WE:Consulting (Consulting)
Send PM
User avatar
Retired Moderator
Joined: 20 Dec 2010
Posts: 1114
Own Kudos [?]: 4702 [1]
Given Kudos: 376
Send PM
Re: Governments have only one response to public criticism of socially nec [#permalink]
1
Bookmarks
crackHSW wrote:
Why is E not the answer , can someone explain ,In E it is mentioned that if no regulation happens then cost willnot increase , which can be inferred from the statement --->Government inevitably make the activity more expensive by regulating it ..... the government is certain to respond. how do u assume that government response can only be by regulation ???


No assumption:
Governments have only one response to public criticism of socially necessary services: regulation of the activity. Thus, regulation will happen, for the public criticism happened.

"E" can't be inferred properly.
If event A occurs, B occurs.
Does not mean:
If event A does not occur, B will not occur.

If I go to the bar, I'll drink Scotch whiskey.
Does not mean;
If I do not go to the bar, I won't drink Scotch whiskey. I may drink one at home, who knows?

B is correct because of the ripple:
A occurs->B occurs->C occurs
We can infer only the following:
If A occurs, B occurs
If A occurs, C occurs
If B occurs, C occurs.

Rest is written in my earlier post.
Manager
Manager
Joined: 15 Sep 2009
Posts: 134
Own Kudos [?]: 121 [0]
Given Kudos: 6
Send PM
Re: Governments have only one response to public criticism of socially nec [#permalink]
Straight B.

The correct answers to inference questions avoid extremes and have no frills: in short, they stick to the script and avoid drawing conclusions for which sufficient info is not provided.

If government is certain to respond to public outcry by regulating day care, then costs will inevitably rise.

Cheers,
Der alte Fritz.
Intern
Intern
Joined: 27 Dec 2014
Posts: 44
Own Kudos [?]: 221 [2]
Given Kudos: 98
Concentration: Leadership, Technology
Send PM
Re: Governments have only one response to public criticism of socially nec [#permalink]
2
Kudos
(A) The quality of child care will improve.
INCORRECT - The passage only talks about governments response. The outcome of the response is not discussed

(B) The cost of providing child-care services will increase.
CORRECT. This is in line with the second sentence of the passage

(C) The government will use funding to foster advances in child care.
INCORRECT. Advancement of child care is no where discussed in the passage

(D) If public criticism of policy is strongly voiced, the government is certain to respond.
INCORRECT. "policy" is not same as "social necessity"

(E) If child-care services are not regulated, the cost of providing child care will not increase
INCORRECT. What will happen without regulation is not in scope of this passage
GMAT Club Legend
GMAT Club Legend
Joined: 03 Oct 2013
Affiliations: CrackVerbal
Posts: 4946
Own Kudos [?]: 7625 [3]
Given Kudos: 215
Location: India
Send PM
Re: Governments have only one response to public criticism of socially nec [#permalink]
2
Kudos
1
Bookmarks
Top Contributor
Simplify the argument -

Governments : Public Criticism of Socially necessary services --> regulation of those activities

Regulating Activities --> makes them more expensive.

Combing the two statements we get -
Public Criticism of socially necessary services --> regulation of those services --> services become more expensive

Childcare services = socially necessary services.

Public Criticism of childcare services --> government is certain to respond --> regulation of childcare services (Note that this is the only response governments have) --> childcare services become more expensive.

Which answer option states this? Option B.

Option A - Incorrect.
We cannot infer anything about "quality" as it is not mentioned in the argument.

Option B - Correct Answer.

Option C - Incorrect.
We have no idea whether the government will do this or not as there is no mention of "funding" or "fostering advances".

Option D - Incorrect.
We cannot make this generalization. We know only about childcare services -
"since public criticism of childcare services has undermined all confidence in such services, and since such services are socially necessary, the government is certain to respond."
Also, here we need the service to be "socially necessary", not just the criticism to be "strongly voiced".

Option E - Incorrect.
We cannot make this inference. We know that "regulating it" will increase the cost; we do not what effects not regulating such services will have.
Current Student
Joined: 14 Nov 2016
Posts: 1174
Own Kudos [?]: 20706 [0]
Given Kudos: 926
Location: Malaysia
Concentration: General Management, Strategy
GMAT 1: 750 Q51 V40 (Online)
GPA: 3.53
Send PM
Re: Governments have only one response to public criticism of socially nec [#permalink]
gurpreet07 wrote:
Governments have only one response to public criticism of socially necessary services: regulation of the activity of providing those services. But governments inevitably make the activity more expensive by regulating it, and that is particularly troublesome in these times of strained financial resources. However, since public criticism of child-care services has undermined all confidence in such services, and since such services are socially necessary, the government is certain to respond.

Which one of the following statements can be inferred from the passage?

(A) The quality of child care will improve.
(B) The cost of providing child-care services will increase.
(C) The government will use funding to foster advances in child care.
(D) If public criticism of policy is strongly voiced, the government is certain to respond.
(E) If child-care services are not regulated, the cost of providing child care will not increase.

Source : LSAT PrepTest 2 - October 1991 Q9


Quote:
Public criticism of service --> regulation --> increased cost

Child care is publicly criticized --> there will be regulation

OKAY. So we can feel pretty confident that the correct answer is going to say something about cost.

(A) This is actually a pretty tricky answer, because the whole point of regulation is to improve the service. But we can't definitively say it'll happen.

(B) CORRECT. Yep. We know regulation leads to increased cost, and we know there will be regulation.

(C) This comes totally out of nowhere. Wuzzah?

(D) We don't have to infer this, because it's basically been stated already.

(E) This is what we call "reversed logic". Remember if A-->B, that doesn't mean -A-->-B (the correct contrapositive would be -B-->-A, the meaning of which would be "If the cost of a service hasn't gone up, it hasn't been subjected to regulation.")
Current Student
Joined: 14 Nov 2016
Posts: 1174
Own Kudos [?]: 20706 [0]
Given Kudos: 926
Location: Malaysia
Concentration: General Management, Strategy
GMAT 1: 750 Q51 V40 (Online)
GPA: 3.53
Send PM
Re: Governments have only one response to public criticism of socially nec [#permalink]
gurpreet07 wrote:
Governments have only one response to public criticism of socially necessary services: regulation of the activity of providing those services. But governments inevitably make the activity more expensive by regulating it, and that is particularly troublesome in these times of strained financial resources. However, since public criticism of child-care services has undermined all confidence in such services, and since such services are socially necessary, the government is certain to respond.

Which one of the following statements can be inferred from the passage?

(A) The quality of child care will improve.
(B) The cost of providing child-care services will increase.
(C) The government will use funding to foster advances in child care.
(D) If public criticism of policy is strongly voiced, the government is certain to respond.
(E) If child-care services are not regulated, the cost of providing child care will not increase.

Source : LSAT PrepTest 2 - October 1991 Q9


Must Be True-SN. The correct answer choice is (B)

This stimulus contains several statements that invite inferences. In the first sentence, we see that public criticism of a socially necessary service leads to government regulation. The third sentence just applies this to child care services: because child care services are a socially necessary service and are publicly criticized, they will be regulated.

Answer Choices (A) is wrong because nothing in the stimulus addresses improving the quality of services. (A) assume that government intervention improves the quality. This may or may not be true.

Answer Choice (B): This is the correct answer choice. The second sentence in the stimulus contains a broad statement: when an activity is regulated by the government, it becomes more expensive. We can apply this principle to what we learned from the third statement and determine that childcare services will become more expensive.

Answer choice (C) is wrong because nothing in the stimulus addresses improving the quality of services. This answer choice assumes that government intervention improves the quality. This may or may not be true.

Answer Choice (D), at first glance, seems like a good answer choice because it is consistent with the first sentence. However, the first sentence only talks about socially necessary services, not policy in general. (D), then, goes too far.

Answer Choice (E) makes a logical error, a mistaken negation: we know that if child-care services are regulated, the cost will increase. But, we do not know anything if they are not regulated. Child care costs might increase anyway for some reason completely unrelated to regulation.
Manager
Manager
Joined: 30 Jul 2014
Status:MBA Completed
Affiliations: IIM
Posts: 91
Own Kudos [?]: 97 [0]
Given Kudos: 107
Send PM
Re: Governments have only one response to public criticism of socially nec [#permalink]
Dear Experts, please help - If I can not infer D, then how can I infer A that is eventually dependent on the inference mentioned in option D. How to limit ourselves about what not to infer and what to infer?
GMAT Club Verbal Expert
Joined: 13 Aug 2009
Status: GMAT/GRE/LSAT tutors
Posts: 6917
Own Kudos [?]: 63649 [1]
Given Kudos: 1773
Location: United States (CO)
GMAT 1: 780 Q51 V46
GMAT 2: 800 Q51 V51
GRE 1: Q170 V170

GRE 2: Q170 V170
Send PM
Re: Governments have only one response to public criticism of socially nec [#permalink]
1
Kudos
Expert Reply
DAakash7 wrote:
Dear Experts, please help - If I can not infer D, then how can I infer A that is eventually dependent on the inference mentioned in option D. How to limit ourselves about what not to infer and what to infer?

Quote:
(D) If public criticism of policy is strongly voiced, the government is certain to respond.

The passage does not say that the government is certain to respond to public criticism of any policy, only to public criticism of socially necessary services. Since choice (D) does not specify the type of policy, it cannot be properly inferred.
Manager
Manager
Joined: 24 Mar 2015
Status:love the club...
Posts: 220
Own Kudos [?]: 112 [0]
Given Kudos: 527
Send PM
Re: Governments have only one response to public criticism of socially nec [#permalink]
mikemcgarry wrote:
gurpreet07 wrote:
Governments have only one response to public criticism of socially necessary services: regulation of the activity of providing those services. But governments inevitably make the activity more expensive by regulating it, and that is particularly troublesome in these times of strained financial resources. However, since public criticism of child-care services has undermined all confidence in such services, and since such services are socially necessary, the government is certain to respond.

Which one of the following statements can be inferred from the passage?
(A) The quality of child care will improve.
(B) The cost of providing child-care services will increase.
(C) The government will use funding to foster advances in child care.
(D) If public criticism of policy is strongly voiced, the government is certain to respond.
(E) If child-care services are not regulated, the cost of providing child care will not increase
.

The LSAT arguments are typically harder than GMAT CR arguments, so if you have run out of challenging GMAT CR practice questions, the LSAT questions can be an excellent source of further practice.

This argument is a complex syllogism.
General case: public complains --- government forced to regulate ---- more money spent on the necessary social service.
Then, we apply this general case scenario the specific example of child care
folks are complaining about child care --- the government will have to regulate it --- then it will cost more money to run the child care programs

Now, let's look at the answers:
(A) The quality of child care will improve.
We know that it's quite likely that the government will spend more money on child care to regulate it. Does this necessarily mean that the quality of the child care programs will improve. Perhaps in a magical fairyland, anything at which the government throws money automatically improves, but in the real world, there are oh-so-many examples of the best intentions of government spending going astray, and programs not improving despite increased spending. It's very helpful to have a real world sense of these things ---- both GMAT CR and LSAT argument questions are consistent with the push & pull of real world forces.
(A) is incorrect.

(B) The cost of providing child-care services will increase.
This is the direct result of the parallel logic. Notice --- we don't know to whom the costs increase: will that increased cost be paid by the parents who want childcare, or by the government (i.e. by all taxpayers)? We don't know, but this is irrelevant. Regardless of who pays, the cost will be increased. (B) is a promising and plausible answer.

(C) The government will use funding to foster advances in child care.
Similar to (A). Yes, the government is going to through money at the programs in the form of regulating it, but will these regulations produce radically improved programs? Of course, that's the idealized hope, but there's no guarantee that this will be the outcome.
(D) is incorrect.

(D) If public criticism of policy is strongly voiced, the government is certain to respond.
Very tricky. This passage is very specifically about "socially necessary services". The government funds a whole bunch of things that are not in the category of socially necessary services (the arts, environmental protection, diplomacy & foreign aid, etc. etc.) Will the government respond to strong criticism about any of these? We don't know. It's well outside the scope of this argument.
(D) is incorrect.

(E) If child-care services are not regulated, the cost of providing child care will not increase
This is the one about which smartmanav asked. Technically, this answer choice is relying on a logical mistake known as the equivalence of a conditional statement and its inverse. In formal language, the mistake is
Starting with (if P, then Q), we conclude (if not P, then not Q).
This is a logical fallacy.
Consider these examples:
Original: If I am in the SF Bay Area, then I am in California. True
Inverse: If I am not in the SF Bay Area, then I am not in California. False --- I could be in LA or San Diego or Tahoe or Yosemite or etc. California is a big state!

Original: If a shape is a square, then it is a quadrilateral. True
Inverse: If a shape is not a square, then it is not a quadrilateral. False --- it could be a trapezoid, a slanted rhombus, an elongated rectangle, etc. etc.

The argument makes the case that
If government regulates, then costs increase.
For the purposes of this question, we have to accept the prompt evidence as true.
Essentially, answer choice (E) leaps from here to the inverse:
If government does not regulate, then costs will not increase.
If the government does not regular child care, that particular source of cost increase is eliminated, but that is certainly not a guarantee that nothing else at all will cause the price to increase. There could be a labor dispute, lawsuits from parents, insurance increases, etc. etc. --- any one of a thousand other things that could cause the price of child care to increase. We don't have a guarantee that the cost of child care will increase, but we certainly can't conclude definitively that the cost won't increase. The truth is: if the government does not regular child care, we have absolutely no idea whether the cost of child care will increase or not. We can draw no conclusion. We have absolutely no grounds for concluding the inverse from the original conditional statement ---- this is a fundamental logical fallacy.
(E) is incorrect.

The only possible answer is (B).

Does all of this make sense?

Mike :-)



hi mikemcgarry

wonderful was your explanation to discard E. Thank you! :-)

many a time, however, I have seen such logical fallacy in place to strengthen an argument. Specifically - although not pertaining to this question - is the usage of such fallacy a legitimate way of strengthening any argument ...?

thanks in advance, man 8-)
Magoosh GMAT Instructor
Joined: 28 Dec 2011
Posts: 4448
Own Kudos [?]: 28569 [1]
Given Kudos: 130
Re: Governments have only one response to public criticism of socially nec [#permalink]
1
Kudos
Expert Reply
gmatcracker2017 wrote:
hi mikemcgarry

wonderful was your explanation to discard E. Thank you! :-)

many a time, however, I have seen such logical fallacy in place to strengthen an argument. Specifically - although not pertaining to this question - is the usage of such fallacy a legitimate way of strengthening any argument ...?

thanks in advance, man 8-)

Dear gmatcracker2017,

I'm happy to respond. :-)

Can one advance a logical fallacy as a legitimate way to strengthen an argument? On the GMAT CR, absolutely not. The GMAT is a magical world in which everything is perfectly logical, so a logical fallacy would carry no weight at all. ON the GMAT CR, nothing good can come from a logical fallacy.

That's on the GMAT. In politics and in the business world, you will at times run into people, especially people in power, who simply want to believe whatever they believe, and if an argument for what they believe needs strengthening, a logical fallacy is just as good as anything else to support it. It's sad, but the real world falls astoundingly short of the high standard of logic and intellectual rigor found on the GMAT.

Does all this make sense?
Mike :-)
Verbal Forum Moderator
Joined: 08 Dec 2013
Status:Greatness begins beyond your comfort zone
Posts: 2101
Own Kudos [?]: 8808 [0]
Given Kudos: 171
Location: India
Concentration: General Management, Strategy
GPA: 3.2
WE:Information Technology (Consulting)
Send PM
Re: Governments have only one response to public criticism of socially nec [#permalink]
gurpreet07 wrote:
Governments have only one response to public criticism of socially necessary services: regulation of the activity of providing those services. But governments inevitably make the activity more expensive by regulating it, and that is particularly troublesome in these times of strained financial resources. However, since public criticism of child-care services has undermined all confidence in such services, and since such services are socially necessary, the government is certain to respond.

Which one of the following statements can be inferred from the passage?

(A) The quality of child care will improve.
(B) The cost of providing child-care services will increase.
(C) The government will use funding to foster advances in child care.
(D) If public criticism of policy is strongly voiced, the government is certain to respond.
(E) If child-care services are not regulated, the cost of providing child care will not increase.

Source : LSAT PrepTest 2 - October 1991 Q9


Public criticism of service --> regulation --> increased cost

Child care is publicly criticized --> there will be regulation

OKAY. So we can feel pretty confident that the correct answer is going to say something about cost.

(A) This is actually a pretty tricky answer, because the whole point of regulation is to improve the service. But we can't definitively say it'll happen.

(B) CORRECT. Yep. We know regulation leads to increased cost, and we know there will be regulation.

(C)Irrelevant

(D) We only can infer this for socially necessary services

(E) This is what we call "reversed logic". Remember if A-->B, that doesn't mean -A-->-B (the correct contrapositive would be -B-->-A, the meaning of which would be "If the cost of a service hasn't gone up, it hasn't been subjected to regulation." ).
Senior Manager
Senior Manager
Joined: 05 Jul 2017
Posts: 458
Own Kudos [?]: 723 [0]
Given Kudos: 294
Location: India
GMAT 1: 700 Q49 V36
GPA: 4
Send PM
Re: Governments have only one response to public criticism of socially nec [#permalink]
hey nightblade354

Although i got this one right within a min, I am confused with the usage of 'however' in the last sentence of the premise. i didn't understand the need to use however and also the need for the contrast

Can you put some light on this?
Current Student
Joined: 31 Jul 2017
Status:He came. He saw. He conquered. -- Going to Business School -- Corruptus in Extremis
Posts: 1734
Own Kudos [?]: 5734 [1]
Given Kudos: 3054
Location: United States (MA)
Concentration: Finance, Economics
Send PM
Re: Governments have only one response to public criticism of socially nec [#permalink]
1
Kudos
Expert Reply
pikolo2510 wrote:
hey nightblade354

Although i got this one right within a min, I am confused with the usage of 'however' in the last sentence of the premise. i didn't understand the need to use however and also the need for the contrast

Can you put some light on this?


My guess is that it would be something about cost. That although it will raise prices, it is needed because of public criticism, but they do not want raised costs. People complain, but will complain about higher prices, maybe? I agree that it is a little weird, but fairly straightforward if you understand the conclusion.
Intern
Intern
Joined: 12 Jul 2021
Posts: 39
Own Kudos [?]: 2 [0]
Given Kudos: 11
Location: India
GMAT 1: 700 Q49 V35
Send PM
Re: Governments have only one response to public criticism of socially nec [#permalink]
Can someone please explain why is D wrong.

Posted from my mobile device
CEO
CEO
Joined: 07 Mar 2019
Posts: 2552
Own Kudos [?]: 1813 [1]
Given Kudos: 763
Location: India
WE:Sales (Energy and Utilities)
Send PM
Re: Governments have only one response to public criticism of socially nec [#permalink]
1
Kudos
AlexSgmat wrote:
Can someone please explain why is D wrong.

Posted from my mobile device

I would request you to go through above explanations of experts(GMATNInja/Mikemcgarry). If you have already gone through it, let me rephrase it.

(D) If public criticism of policy is strongly voiced, the government is certain to respond.

Which policy is D referring to?
In the passage it was about socially necessary services. But this choice expands the scope of that policy to others also, which we are not sure of. That is the specific reason this choice is wrong. It makes a generic statement which may or may not come true.

HTHs.
GMAT Club Verbal Expert
Joined: 13 Aug 2009
Status: GMAT/GRE/LSAT tutors
Posts: 6917
Own Kudos [?]: 63649 [0]
Given Kudos: 1773
Location: United States (CO)
GMAT 1: 780 Q51 V46
GMAT 2: 800 Q51 V51
GRE 1: Q170 V170

GRE 2: Q170 V170
Send PM
Re: Governments have only one response to public criticism of socially nec [#permalink]
Expert Reply
AlexSgmat wrote:
Can someone please explain why is D wrong.

Posted from my mobile device

Check out this post and let us know if you still have questions!
Director
Director
Joined: 11 Sep 2022
Posts: 501
Own Kudos [?]: 151 [0]
Given Kudos: 2
Location: India
Paras: Bhawsar
GMAT 1: 590 Q47 V24
GMAT 2: 580 Q49 V21
GMAT 3: 700 Q49 V35
GPA: 3.2
WE:Project Management (Other)
Send PM
Re: Governments have only one response to public criticism of socially nec [#permalink]
From the passage, the following statement can be inferred:

(B) The cost of providing child-care services will increase.

The passage mentions that government regulation of socially necessary services makes those services more expensive. Since child-care services are considered socially necessary, it can be inferred that the cost of providing child-care services will increase if the government responds to public criticism by regulating them.
GMAT Club Bot
Re: Governments have only one response to public criticism of socially nec [#permalink]
Moderators:
GMAT Club Verbal Expert
6917 posts
GMAT Club Verbal Expert
238 posts
CR Forum Moderator
832 posts

Powered by phpBB © phpBB Group | Emoji artwork provided by EmojiOne